Drug company manager: Our newest product is just not selling. One way to save it would be a new marketing campaign. T...

Louis on January 30, 2017

Not sure why D

Please Explain. Thanks.

Replies
Create a free account to read and take part in forum discussions.

Already have an account? log in

Mehran on February 1, 2017

@Louis the drug company manager's conclusion is, "so we should try it."

Try what? A new marketing campaign for the company's newest product that is just not selling.

The support? While this would not guarantee success, it is one chance to save the product.

This is a Weaken question, so we are looking for an answer choice that weakens the drug company's manager's argument that the company should try a new marketing campaign.

(D) states, "Undertaking a new marketing campaign would endanger the drug company's overall position by necessitating cutbacks in existing marketing campaigns."

This clearly weakens the argument by pointing out that such a new marketing campaign would endanger the drug company's overall position.

So not only will the new marketing campaign campaign not guarantee success of its newest product, but it would also endanger the company as whole.

Therefore, (D) would be the correct answer.

Hope that helps! Please let us know if you have any other questions.

meisen on June 1, 2018

Why is B wrong?

Mehran on June 2, 2018

Hey @meisen, thanks for your post.

As explained above, on this Weaken question, you are looking for the answer choice that weakens the drug company's manager's argument that the company should try a new marketing campaign because doing so is one chance to save the product.

The drug company manager does not say that a new marketing campaign is *guaranteed* to save the new product. If he had, then (B) might be a viable answer choice. But since the drug company manager acknowledges that the new marketing campaign will not guarantee success, the fact set out in (B) - that many new products fail whether or not supported by marketing campaigns - is not going to weaken the argument in the stimulus.

By contrast, and as explained above, the fact set out in answer choice (D) does weaken the argument in the stimulus.

Ohemaa on June 20, 2018

Why is A wrong?

Anita on June 21, 2018

@Ohemaa I actually see A as strengthening the argument if anything, which is why it can be enticing. If the losses due to the product would be a negative, that could support the claim that we should do the marketing campaign to save it. But we're looking for a weaken here, so A we can rule out.